Hi could I get some assistance?

start by sketching the functions

I see that the region of interest is in quadrant I ...
Finding the area will require two integrals if done w/respect to x and only one integral w/respect to y.

Also note the intersection coordinates of the quadratic curve and the line is not numerically “nice”.

show us what you can come up with on your own to start
 
Please understand that by help we on this forum do not equate that with doing the problem for you.
This is a free math help forum and not a free homework service forum.

Fair enough you never asked for anyone to do the problem for you but not showing any work seems to suggest that.

If you had read the forum's posting guidelines you would know that to receive help you need to share the work you have done so far, even if you know it is wrong. This way we get to see what method you want to use to solve the problem and where you are making any mistakes. Then we know exactly what type of help to offer. So please post back!
 
Top